Diskussion:Lense-Thirring-Effekt

aus Wikipedia, der freien Enzyklopädie
Zur Navigation springen Zur Suche springen

Quelle der Erstfassung: Nature 431(2004),958 --มีชา disk. 14:54, 14. Dez 2004 (CET)

Rotieren oder irgendwie bewegen?[Quelltext bearbeiten]

Hi!

Meines Erachtens tritt der Effekt auch auf, wenn sich eine Masse irgendwieanders zum Beobachter bewegt, also eine Galaxie an mir vorbeifliegt z.B. Rotation muss nicht unbedingt vorhanden sein, oder? --Szs 11:27, 8. Nov 2005 (CET)

Das gravitomagnetische Feld beschreibt die Umgebung rotierender Massen und verschwindet bei nicht rotierenden, statischen Massen. "Frame-Dragging besagt, dass jede rotierende Masse das lokale Bezugssystem (engl. reference frame) mitzieht (engl. 'ziehen': to drag). Anders gesagt wird alles von der rotierenden Masse gezwungen sich mit der Drehrichtung der Masse zu drehen."Quelle: http://www.wissenschaft-online.de/astrowissen/lexdt_f04.html#fra --91.34.201.122 13:20, 3. Jun. 2014 (CEST)[Beantworten]
Effekte der Gravitation treten auch auf wenn die Galaxie einfach vorbeifliegt, aber die werden dann nicht als Lense-Thirring-Effekt bezeichnet. --mfb (Diskussion) 15:36, 3. Jun. 2014 (CEST)[Beantworten]

Quantifizierung[Quelltext bearbeiten]

bei Boyer-Lindquist sind die Parameter a (Kerr-Parameter) und α (Lapse-Funktion) zu unterscheiden, daher sollte hier a verwendet werden. Ra-raisch (Diskussion) 11:33, 3. Jul. 2016 (CEST) erledigtErledigt Ra-raisch (Diskussion) 13:26, 13. Sep. 2017 (CEST)[Beantworten]

Erklärung des Effekts[Quelltext bearbeiten]

Mir fehlt im Artikel (von der kurzen Einleitung einmal abgesehen) eine genauere Beschreibung und Erklärung des Effekts. Dass der erste Absatz des Artikels sich schon mit einem experimentellen Nachweis beschäftigt, nach dem nur grob umrissen wurde was es überhaupt nachzuweisen gilt scheint mir sehr merkwürdig. --77.8.163.250 02:37, 25. Okt. 2016 (CEST)[Beantworten]

Geht mir genauso. --Hfst (Diskussion) 08:20, 20. Apr. 2019 (CEST)[Beantworten]
Die Erklärung des Effekts ist relativ einfach, es ist ein Effekt der Theorie. Würde man bereits vorab die Bewegung richtig beschreiben, gebe es diesen Effekt nicht.
Er ist also eine Korrektur einer fehlerhaften Beschreibung. In der bis dahin gemachten, fehlerhaften, Beschreibung tritt er nicht auf.
Er existier daher nur relativ zu der bis dahin bekannten Beschreibung, er ist also die Korrektur einer bis dahin bekannten fehlerhaften Theorie.
Der Effekt bedeutet das selbst eine Kreisbahn eine "Periheldrehung" hat, weil eben das reale Potential nicht newtonsch ist. Auch die Drehung des Zentralkörpers erzeugt ein zusätzliches Potential, welches eine zusätzliche Bewegung bewirkt. Insofern wird da selbstverständlich nichts mitgezogen, so eine Erklärung ist Okkultismus, denn dazu gibt es kein Wirkprinzip, zumindest kennt die ART kein Wirkprinzip dazu. (nicht signierter Beitrag von 46.223.163.57 (Diskussion) 18:36, 5. Dez. 2019 (CET))[Beantworten]

anstatt sollte man doch das meist übliche benützen, wenn auch noch das "M" unterschlagen wurde. Ra-raisch (Diskussion) 13:29, 13. Sep. 2017 (CEST)[Beantworten]

Im Artikel wird wiederholt die Formulierung "Verdrillung" verwendet. Es stellt sich die Frage, ob diese Formulierung eine falsche Anschauung unterstützt - bis hin zu der Vorstellung, dass eine verdrillte Raumzeit wieder entdrillt werden kann? In der englischen Literatur finden sich eher Formulierungen wie "frame dragging" und "everything is swept along", wodurch die Vorstellung des Verdrillens nicht gefördert wird. Erg (Diskussion) 02:47, 9. Jul. 2020 (CEST)[Beantworten]

Wenn die deutschsprachige Literatur diesen Begriff verwendet, dann macht WP das auch, egal ob der Begriff treffend ist oder nicht. Siehe WP:Begriffsfindung. --Hfst (Diskussion) 08:17, 9. Jul. 2020 (CEST)[Beantworten]
Das ist mir schon klar. Ich stelle in Frage, dass das durch deutschsprachige Literatur ausreichend gedeckt ist. Man findet zwar durch eine oberflächliche Websuche häufiger den Begriff der Verdrillung, aber das dürfte kein ausreichender Beleg sein. Erg (Diskussion) 16:51, 9. Jul. 2020 (CEST)[Beantworten]

Fehler in „Genauere Formulierung“?[Quelltext bearbeiten]

Ich lese den ersten Satz

„Die Rotationswinkelgeschwindigkeit ω des Raumes um eine […] Masse mit dem Spinparameter a […] ergibt sich in Boyer-Lindquist-Koordinaten mit G = M = c = K = 1 […] mit
 Ω = […]
 mit den Termen
 […]“

und erwarte eine Formel für die Rotationswinkelgeschwindigkeit ω. Statt dessen finde ich eine Formel für Ω, in der nirgendwo ein ω auftaucht. Sollte statt des großen Omega ein kleines stehen? Sollte es nicht besser statt „ergibt sich mit“ heißen: „ergibt sich zu“? Sollte der Satz nicht mit einem Punkt enden? Sollten zwischen den Termen nicht Kommata stehen? Was erklärt Σ, und warum taucht der Term gar nicht in der Gleichung auf? Fragen des verwunderten Laien --Joachim Schnitter (Diskussion) 05:17, 4. Jan. 2022 (CET)[Beantworten]

"großes Omega" ist "doppelt-gemoppelt". "O mega" heißt bereits "großes O", das kleine o dagegen, das auch zur Bezeichnung einer Covid-Variante verwendet wird, "O mikron". --2A04:4540:1504:CA00:8502:605D:71D1:F843 18:14, 15. Feb. 2022 (CET)[Beantworten]